Polynomial
by Z_., Apr 23, 2025, 9:21 PM
Let
be an integer greater than zero. Then, the value of the sum of the reciprocals of the cubes of the roots of the equation
is equal to:

![\[
mx^4 + 8x^3 - 139x^2 - 18x + 9 = 0
\]](http://latex.artofproblemsolving.com/a/3/a/a3a9bca412e927632e4921c51f2a6593409031d9.png)
This post has been edited 1 time. Last edited by Z_., 3 hours ago
Reason: .
Reason: .
Inequalities
by Scientist10, Apr 23, 2025, 6:36 PM
interesting function equation (fe) in IR
by skellyrah, Apr 23, 2025, 9:51 AM
find all function F: IR->IR such that 

Tangents forms triangle with two times less area
by NO_SQUARES, Apr 23, 2025, 9:08 AM
Let
be triangle, inscribed in parabola. Tangents in points
forms triangle
. Prove that
. (
is area of triangle
).
From F.S.Macaulay's book «Geometrical Conics», suggested by M. Panov






From F.S.Macaulay's book «Geometrical Conics», suggested by M. Panov
Existence of perfect squares
by egxa, Apr 18, 2025, 9:48 AM
Find all natural numbers
for which there exists an even natural number
such that the number
is a perfect square.


![\[
(a - 1)(a^2 - 1)\cdots(a^n - 1)
\]](http://latex.artofproblemsolving.com/9/c/a/9caf4eeb82ff46b5ba55ab4b6bc28f0cace586ec.png)
FE solution too simple?
by Yiyj1, Apr 9, 2025, 3:26 AM
Find all functions
such that the equality
holds for all pairs of real numbers
.
My solution
I feel like my solution is too simple. Is there something I did wrong or something I missed?



My solution
Clearly,
is an obvious solution. Now, let
. Then, we have
or
. Therefore, the solutions are
.





I feel like my solution is too simple. Is there something I did wrong or something I missed?
Number Theory
by fasttrust_12-mn, Aug 16, 2024, 10:21 AM
Find all integers
for which
is the square of an integer


Floor double summation
by CyclicISLscelesTrapezoid, Jul 12, 2022, 12:52 PM
Which positive integers
make the equation
true?

![\[\sum_{i=1}^n \sum_{j=1}^n \left\lfloor \frac{ij}{n+1} \right\rfloor=\frac{n^2(n-1)}{4}\]](http://latex.artofproblemsolving.com/e/7/f/e7fb8c20a43535b5aaed5ab254f1ef043263c62b.png)
IMO 2015 Problem 3
by liberator, Jul 14, 2015, 2:48 PM
Problem: Let
be an acute triangle with
. Let
be its cirumcircle,
its orthocenter, and
the foot of the altitude from
. Let
be the midpoint of
. Let
be the point on
such that
and let
be the point on
such that
. Assume that the points
,
,
,
and
are all different and lie on
in this order.
Prove that the circumcircles of triangles
and
are tangent to each other.
Proposed by Ukraine
My solution![[asy]
/* IMO 2015 Problem 3, free script by liberator, 14 July 2015 */
import olympiad;
import cse5;
unitsize(3.2cm);
pen ddd = rgb(0.4,0.6,0.8);
pointpen=black;
defaultpen(fontsize(10pt));
pair A = dir(75);
pair B = dir(205);
pair C = dir(-25);
pair O = origin;
pair H = orthocenter(A,B,C);
pair F = foot(A,B,C);
pair D = IP(L(H,F,0,2015),unitcircle);
pair M = midpoint(B--C);
pair Ap = B+C-H;
pair Q = IP(L(M,H,0,2015), unitcircle);
pair Qp = A+Ap-Q;
pair K = IP(L(Qp,H,-0.1,2015), unitcircle);
pair P = circumcenter(D,H,K);
D(unitcircle, heavygreen);
D(circumcircle(K,Q,H), red+linewidth(1));
D(circumcircle(F,K,M), heavymagenta);
D(circumcircle(D,H,K), grey+linetype("0 2")+linewidth(1));
D(A--B--C--cycle, ddd+linewidth(1));
D(A--D, ddd);
D(A--Qp, ddd);
D(Q--Ap, ddd+dashed);
D(K--Qp, ddd+dashed);
D(H--P--K, ddd);
D(rightanglemark(C,F,A,2), ddd);
D("A",A,dir(A)*2);
D("B",B,dir(B));
D("C",C,dir(C));
D("D",D,dir(-110));
D("F",F,dir(F));
D("H",H,dir(-65)*2);
D("K",K,dir(-15)*1.6);
D("M",M,dir(-70));
D("P",P,dir(P));
D("Q",Q,dir(Q));
D("A'",Ap,dir(Ap));
D("Q'",Qp,dir(Qp));
D("\Gamma",O,dir(160)*28);
[/asy]](//latex.artofproblemsolving.com/a/a/6/aa6ae4ba390a833b56195faffc62aaf837297443.png)
Let
be the antipodes of
respectively w.r.t
, and let
be the reflection of
in
.
is a Miquel point, so
are collinear. Also observe that
are collinear, since
.
Let
be the circumcenter of
, which lies on the perpendicular bisector
of
: we claim that
. By a converse of Reim's theorem, the tangent
to
at
is parallel to
; since
from rectangle
, it follows that
, and our claim has been established.
Now
, so
is tangent to
. On the other hand,
, so
is tangent to
: since
,
is also tangent to
, and the result follows.




















Prove that the circumcircles of triangles


Proposed by Ukraine
My solution
![[asy]
/* IMO 2015 Problem 3, free script by liberator, 14 July 2015 */
import olympiad;
import cse5;
unitsize(3.2cm);
pen ddd = rgb(0.4,0.6,0.8);
pointpen=black;
defaultpen(fontsize(10pt));
pair A = dir(75);
pair B = dir(205);
pair C = dir(-25);
pair O = origin;
pair H = orthocenter(A,B,C);
pair F = foot(A,B,C);
pair D = IP(L(H,F,0,2015),unitcircle);
pair M = midpoint(B--C);
pair Ap = B+C-H;
pair Q = IP(L(M,H,0,2015), unitcircle);
pair Qp = A+Ap-Q;
pair K = IP(L(Qp,H,-0.1,2015), unitcircle);
pair P = circumcenter(D,H,K);
D(unitcircle, heavygreen);
D(circumcircle(K,Q,H), red+linewidth(1));
D(circumcircle(F,K,M), heavymagenta);
D(circumcircle(D,H,K), grey+linetype("0 2")+linewidth(1));
D(A--B--C--cycle, ddd+linewidth(1));
D(A--D, ddd);
D(A--Qp, ddd);
D(Q--Ap, ddd+dashed);
D(K--Qp, ddd+dashed);
D(H--P--K, ddd);
D(rightanglemark(C,F,A,2), ddd);
D("A",A,dir(A)*2);
D("B",B,dir(B));
D("C",C,dir(C));
D("D",D,dir(-110));
D("F",F,dir(F));
D("H",H,dir(-65)*2);
D("K",K,dir(-15)*1.6);
D("M",M,dir(-70));
D("P",P,dir(P));
D("Q",Q,dir(Q));
D("A'",Ap,dir(Ap));
D("Q'",Qp,dir(Qp));
D("\Gamma",O,dir(160)*28);
[/asy]](http://latex.artofproblemsolving.com/a/a/6/aa6ae4ba390a833b56195faffc62aaf837297443.png)
Let










Let












Now









This post has been edited 2 times. Last edited by liberator, Jul 14, 2015, 8:58 PM
Coaxal circles in incenter/excenter configuration
by liberator, Apr 15, 2015, 7:18 PM
Problem: Let
be a triangle, whose excircle (opposite
) touches
at
respectively. Denote
as the intersection of the lines
and
, and
as the intersection of the lines
and
. If
is the excenter of
, opposite
, prove that the circumcircles of triangles
and
are coaxal.
Commentary: We may replace "excircle" with "incircle", "excenter" with "incenter", and the result still holds.
See interactive diagram here.
My solution















Commentary: We may replace "excircle" with "incircle", "excenter" with "incenter", and the result still holds.
See interactive diagram here.
My solution
Let
denote the second intersection of circles
and
. It suffices to show that
is on
.
First, observe that
is cyclic, from perpendicularity of tangents.
We claim that
is tangent to
at
and
at
. Let
meet
again at
. By Reim's theorem on
and
, we have
. Then by Reim's theorem again, on
and
, the two circles are tangent at
. Similarly,
is tangent to
at
, which establishes our claim.
Now, let
be the radical center of the three circles
. Being the intersection of the two tangents to
at
and
,
is on
, the perpendicular bisector of
. Hence
, so
is on
, as required.





First, observe that

We claim that

















Now, let











IMO 2014 Problem 4
by ipaper, Jul 9, 2014, 11:38 AM
Let
and
be on segment
of an acute triangle
such that
and
. Let
and
be the points on
and
, respectively, such that
is the midpoint of
and
is the midpoint of
. Prove that the intersection of
and
is on the circumference of triangle
.
Proposed by Giorgi Arabidze, Georgia.

















Proposed by Giorgi Arabidze, Georgia.
Archives







Shouts
Submit
56 shouts
Tags
About Owner
- Posts: 95
- Joined: May 28, 2014
Blog Stats
- Blog created: Aug 13, 2014
- Total entries: 46
- Total visits: 38185
- Total comments: 43
Search Blog